User Avatar
IrvinPhan
Joined
Jun 2025
Subscription
Live
PrepTests ·
PT127.S3.Q12
User Avatar
IrvinPhan
Thursday, Oct 23 2025

@LazyBrain 100% yes

1
PrepTests ·
PT156.S4.Q10
User Avatar
IrvinPhan
Edited Monday, Sep 29 2025

Almost got my sleep deprived bum until I remembered the word "residents" in the stimulus

0
PrepTests ·
PT158.S2.Q10
User Avatar
IrvinPhan
Monday, Sep 22 2025

@angeliqueantonini01643 PT111 S1 Q17

0
PrepTests ·
PT123.S2.Q15
User Avatar
IrvinPhan
Edited Thursday, Sep 11 2025

@1ongNose before I attempt to answer your question, I would just like to preface that I haven't yet watched the explanation video for this question, so I'm not aware of how J.Y. or Kevin interpreted this question. I'm sure they can do a far better job than I can in terms of explaining this question, as well as any of the other 7Sage tutors.

First, as to how c) is stated in the question stem, we know the following from the stimulus:

Each year's vaccination will protect only against the strain of the influenza virus deemed most likely to be prevalent that year...

And, answer choice c) states:

No vaccine for the influenza virus protects against more than one strain of that virus.

We know, from the stimulus, that the influenza virus will protect only against the strain of the influenza virus deemed most likely to be prevalent that year. Which, we can map into Lawgic as:

  • protect -> most prevalent influenza strain that year

If we get the contrapositive, then we get:

  • /most prevalent influenza strain that year -> /protect

Which, we can interpret as, if you do not contract the most prevalent influenza strain that year, then the vaccine will not protect against it.

Which, from this, we can reasonably infer then that answer choice c) is stated (albeit indirectly), as there can only be one most prevalent strain of influenza virus in a given year, which is the only strain that the influenza vaccine protects against. It doesn't protect against anything else.

Answer choice c) is a necessary assumption, sure. It might be a redundant statement since we already have it in the stimulus, but it is a necessary assumption. If we were to negate it, then the argument would fall flat. Were this a necessary assumption question, I have no doubt in my mind that it would be the correct answer.

The issue however is that we are trying to find a sufficient assumption. And, this is where answer choice c) fails. We need to sufficiently prove our conclusion--we need to make it so that the conclusion is in sufficiency territory. And, knowing answer choice c) doesn't help us on that front, because it doesn't prove the idea that every year, high-risk individuals must receive a vaccine for a different strain of the virus.

And, this is where answer choice d) comes in.

Let's pull down the argument again for convenience's sake:

Each year's vaccination will protect only against the strain of the influenza virus deemed most likely to be prevalent that year, so every year it will be necessary for all high-risk individuals to receive a vaccine for a different strain of the virus.

Our premise is each year, the influenza virus of that year only protects against the influenza strain that is deemed most prevalent that year.

And, our conclusion is that every year, it is required (under this government policy) that all high-risk individuals receive the influenza vaccine for a different strain of the virus.

There's a gap in the argument, from premise to conclusion. Why is it that a new vaccine is required each year for a different strain?

Again, were this a necessary assumption question, answer choice c) would be the answer.

But, why is it that high-risk individual has to receive a new influenza vaccine for a different strain of the virus each year? Why can't they just reuse vaccines?

Say a person gets vaccinated for the flu in 2018 with InfluenzaVirus#19. Why is it that in 2019, they have to get vaccinated for the flu with InfluenzaVirus#20? Why not reuse #19? Why can't the most prominent influenza strain be picked twice in a row?

I think by now, you can identify the gap from premise to conclusion now. The author's assumption--and the gap in the argument--is that there must be a different strain of influenza deemed most prevalent, every year.

And, if that is true? Then the author's conclusion is true.

Which, is what answer choice d) gives us:

Each year the strain of influenza virus deemed most likely to be prevalent will be one that had not previously been deemed most likely to be prevalent.

0
User Avatar
IrvinPhan
Edited Monday, Sep 08 2025

Darn, in the morning I thought I was just really good at Logical Reasoning questions for some reason, but now my analytics is fixed and I can see I still suck at LR 🫠

1
PrepTests ·
PT114.S2.Q17
User Avatar
IrvinPhan
Wednesday, Sep 03 2025

@LSATLOVER Hm, I'm not sure if I agree with your assessment of 7Sage's assessment of answer choice b). I feel as though there is an misunderstanding on your part as to Trent's argument.

Selena stipulated the idea what caused the dinosaurs to die out was a worldwide dust layer, which in turn cooled the Earth beyond the point that dinosaurs (or vegetation that supported them) could adapt. And, it is on this basis that Trent finds issue with Selena's argument--which is that the size of the asteroid crater of the Yucatán peninsula (which Selena points to) is simply not big enough to have produced the dust necessary to result in that worldwide dust layer.

Trent isn't arguing that the asteroid itself did not kill off the dinosaurs (which I assume is how you're interpreting Trent's argument). Trent is arguing, "hey, that dust cloud idea you think killed off the dinosaurs? It's not plausible, because the crater that you point to as evidence just isn't big enough to create the necessary amount of dust for your dust cloud idea."

And, regardless of whether or not there is an implicit "some," "most," or "all" in front of answer choice b), I feel like it doesn't matter. We still get the same result as long as we remember that Trent isn't arguing that the asteroid itself did not kill off the dinosaurs. Any variant of answer choice b) (all, negated all, some, negated some, most, negated most) doesn't touch on the idea that Trent stated, which is that the dust generated by the asteroid--as evidenced by the size of the crater--simply isn't big enough.

We don't need the dinosaurs in the neighboring area of an asteroid's impact zone to survive. Even if they do, that's fine. Because, again--it doesn't touch on the idea of the dust cloud hypothesis stipulated by Selena, which Trent rejects.

but uh, don't take advice from me, i haven't even taken the lsat yet (and my entire comment hinges on if my interpretation of your interpretation of this question is correct or not)

0

Would it be possible to add an option to remove the transparency effect on the "your score" number after submitting a drill?

I just noticed that you can vaguely make out your score despite the transparent effect, which is really grating, as if I can see that it's 100%, I know that all of my answers were right; if I don't see a 100%, I know that one of my answers were wrong; regardless, both options undermines the uncertainty that's supposed to help me think through the questions more carefully during blind review before checking the correct answers.

It would be really helpful if there was an option to completely hide or mask the score until we press the intentionally reveal button.

3
User Avatar

Wednesday, Jun 18 2025

IrvinPhan

Dark mode for new 7Sage

Is there a dark mode setting for the new 7Sage website? I can't seem to find an option to set it to dark mode (or "Night mode with inverted colors" as it was called on classic 7Sage). If there is no setting for this, does 7Sage plan on implementing dark mode at a later date?

5

Confirm action

Are you sure?